Moment of inertia about z-axis in spherical coordinates

Click For Summary
The discussion focuses on calculating the moment of inertia about the z-axis for a solid defined by specific boundaries in spherical coordinates. The initial attempt to convert the integral to cylindrical coordinates was incorrect, and the correct volume element in spherical coordinates should be used. There were also errors in the limits of integration for the variable ρ, as the solid's boundaries were not properly accounted for. The correct setup involves ensuring the moment arm is accurately represented and that the volume element is correctly defined. Ultimately, a clear understanding of the geometric configuration is crucial for solving the problem accurately.
clairez93
Messages
113
Reaction score
0

Homework Statement



Use spherical coordinates to find the moment of inertia about the z-axis of a solid of uniform density bounded by the hemisphere \rho=cos\varphi, \pi/4\leq\varphi\leq\pi/2, and the cone \varphi=4.

Homework Equations



I_{z} = \int\int\int(x^{2}+y^{2})\rho(x, y, z) dV

The Attempt at a Solution



I tried to convert that equation to cylindrical coordinates and got this (k representing density because it's uniform)

I_{z} = k \int^{2\pi}_{0}\int^{\pi/2}_{\pi/4}\int^{1}_{0}\rho^2 sin^{2}\varphi*d\rho*d\varphi*d\theta

Plugged that into my calculator and got:

\frac{k\pi(\pi+2)}{12}

The book answer is:
\frac{k\pi}{192}

What am I doing wrong?
 
Physics news on Phys.org
clairez93 said:

Homework Statement



Use spherical coordinates to find the moment of inertia about the z-axis of a solid of uniform density bounded by the hemisphere \rho=cos\varphi, \pi/4\leq\varphi\leq\pi/2, and the cone \varphi=4.

You mean \varphi = \pi / 4 for the cone.

Homework Equations



I_{z} = \int\int\int(x^{2}+y^{2})\rho(x, y, z) dV

The Attempt at a Solution



I tried to convert that equation to cylindrical coordinates and got this (k representing density because it's uniform)

I_{z} = k \int^{2\pi}_{0}\int^{\pi/2}_{\pi/4}\int^{1}_{0}\rho^2 sin^{2}\varphi*d\rho*d\varphi*d\theta

That doesn't look like cylindrical coordinates to me. But then, why would you want cylindrical coordinates anyway? Assuming that the \rho^2\sin^2(\phi)
is the moment arm, check your spherical coordinate dV.

Plugged that into my calculator and got:

\frac{k\pi(\pi+2)}{12}

The book answer is:
\frac{k\pi}{192}

What am I doing wrong?

Aside from getting the dV wrong, using a calculator?

[Edit] Looking closer your limits for \rho are also wrong.
 
Last edited:
LCKurtz said:
You mean \varphi = \pi / 4 for the cone.
Yes, sorry, typo.



That doesn't look like cylindrical coordinates to me. But then, why would you want cylindrical coordinates anyway? Assuming that the \rho^2\sin^2(\phi)
is the moment arm, check your spherical coordinate dV.

Sorry, typo, I meant spherical coordinates.

I checked and dV should be

\rho^2\sin^2(\phi)d\rho*d\varphi*d\theta

So that should change the integral to:

I_{z} = k \int^{2\pi}_{0}\int^{\pi/2}_{\pi/4}\int^{1}_{0}(\rho^2 sin^{2}\varphi)^{2}*d\rho*d\varphi*d\theta

Aside from getting the dV wrong, using a calculator?

I usually use my calculator to check my setup, then once I know that is right, I go back and evaluate it by hand.

[Edit] Looking closer your limits for \rho are also wrong.

I'm not sure what to do for \rho, I thought since the radius of the hemisphere was 1, then \rho would go from 0 to 1.
 
clairez93 said:
I checked and dV should be

\rho^2\sin^2(\phi)d\rho*d\varphi*d\theta

The sine should not be squared.

So that should change the integral to:

I_{z} = k \int^{2\pi}_{0}\int^{\pi/2}_{\pi/4}\int^{1}_{0}(\rho^2 sin^{2}\varphi)^{2}*d\rho*d\varphi*d\theta

I'm not sure what to do for \rho, I thought since the radius of the hemisphere was 1, then \rho would go from 0 to 1.

Have you drawn a picture of the desired volume? Your sphere is not centered at the origin and its equation isn't \rho = 1.
 
Question: A clock's minute hand has length 4 and its hour hand has length 3. What is the distance between the tips at the moment when it is increasing most rapidly?(Putnam Exam Question) Answer: Making assumption that both the hands moves at constant angular velocities, the answer is ## \sqrt{7} .## But don't you think this assumption is somewhat doubtful and wrong?

Similar threads

Replies
3
Views
2K
  • · Replies 4 ·
Replies
4
Views
1K
  • · Replies 2 ·
Replies
2
Views
2K
  • · Replies 9 ·
Replies
9
Views
2K
  • · Replies 3 ·
Replies
3
Views
3K
  • · Replies 4 ·
Replies
4
Views
2K
  • · Replies 4 ·
Replies
4
Views
1K
Replies
3
Views
1K
  • · Replies 4 ·
Replies
4
Views
2K
  • · Replies 34 ·
2
Replies
34
Views
3K